冗長的

成本最小化F(x)=米我n(X1,X2)+X3F(X)=米一世n(X1,X2)+X3f(x) = min(x_1,x_2) + x_3

  • January 25, 2021

給出以下生產函式,

$ f(X) = min{x_1,x_2} + x_3 $

這裡有一個解決方案https://math.stackexchange.com/questions/605925/constrained-maximization-of-leontif-utility-function-minx-1-x-2,和這個函式類似。但是,它不包括添加劑 $ x_3 $ .

我最初的解決方案嘗試

重新定義生產函式,這樣,

$ f(X) = x_1 + x_3 $ 如果 $ x_2 < x_1 $ 和 $ f(x) = x_2 + x_3 $ 除此以外。這給出了以下拉格朗日;

$ L = p_1 x_1 + p_2 x_2 + p_3 x_3 - \lambda_1(x_1 +x_3 - Q) - \lambda_2(x_2 + x_3 - Q) $

與每個 $ \lambda_i > 0 $ 這兩個約束都具有約束力,因此,

$ Q=x_1+x_3 $ 和 $ Q=x_2 + x_3 $ ; 然後約束減少到

$ x_1 - x_2 = 0 $ 暗示著 $ x_1 = x_2 $ . 但是,這與我對兩者之間的關係所做的初始條件相矛盾 $ x_1 $ 和 $ x_2 $ 因此,產生的生產函式。

我在這裡有點困惑。我應該如何處理這個問題?

雖然在問題中沒有明確說明,但我從您設置的 Langrangian 函式中猜測您打算解決的問題是

$$ \min_{x_1,x_2,x_3} p_1x_1 + p_2x_2 + p_3x_3 \[8pt] h(x_1,x_2,x_3) = \min{x_1,x_2} + x_3\geq z $$

這項任務結合了生產中的完美補充 $ f(x_1,x_2) = \min {x_1,x_2} $ 有完美的替代品 $ g(y_1,y_2) = y_1 + y_2 $ .

一種方法有時是分兩個階段解決問題。直覺地說,與 $ y_1:=\min{x_1,x_2} $ 公司將生產 $ y_1 $ 以單位成本 $ p_{y_1} = p_1+p_2 $ 對於任何級別 $ y_1 $ 生產出來的它永遠不能最小化使用更多的成本 $ x_1 $ 比 $ x_2 $ 或其他方式。因此 $ x_1=x_2 = \min{x_1,x_2} = y_1 $ 以及每個單位的單獨成本 $ y_1 $ 因此 $ p_1+p_2 $ .

接下來,通過生產解決成本最小化

$$ g(y_1,y_2) = y_1 + y_2 = y_1 + x_3 = z $$

與投入價格 $ p_{y_1} $ 和 $ p_{3} $ . 由於因素 $ y_1 $ 和 $ x_3 $ 同等生產力(恆定等於邊際生產力)生產將以最便宜的要素發生,因此成本必須是 $ C(z,p) =\min{p_{y_1},p_3}z = \min{p_1+p_2,p_3}z $ .

我避免使用拉格朗日乘數法,因為生產函式是不可微的。

引用自:https://economics.stackexchange.com/questions/42283